Q3

 
Lisa
Thanks Received: 0
Vinny Gambini
Vinny Gambini
 
Posts: 1
Joined: December 20th, 2010
 
 
 

Question 3

by Lisa Tue Jan 25, 2011 5:43 pm

Have worked this one several times. Not understanding why the answer is C. Possibly incorrect diagramming? Wrong inferences? Help! Thank you!
 
apom22
Thanks Received: 1
Forum Guests
 
Posts: 9
Joined: December 07th, 2010
 
 
 

Re: Question 3

by apom22 Thu Jan 27, 2011 3:43 pm

There really isn't many inferences to make it's a straightforward sequencing games. You just make a chain through the conditions and follow them.

#3

R T V S Y/W W/Y Z Q

W can be the 5th clown to get out of the car
 
apom22
Thanks Received: 1
Forum Guests
 
Posts: 9
Joined: December 07th, 2010
 
This post thanked 1 time.
 
 

Re: Question 3

by apom22 Thu Jan 27, 2011 3:52 pm

post your diagram and we can figure out where you went wrong.

I looked at this question again and this is what I concluded if Z is 7.

1. Only R or Z can be 1 so if Z is 7 R is 1
2. Z>Q rule forces 8 into Q



for spaces, 2, 3, 4, 5, 6 any combination of T-V-S (with the Y after after the V) will work. The W can go anywhere in those spaces.
User avatar
 
bbirdwell
Thanks Received: 864
Atticus Finch
Atticus Finch
 
Posts: 803
Joined: April 16th, 2009
 
 
 

Re: Question 3

by bbirdwell Sat Jan 29, 2011 2:20 pm

Sounds good, apom! And I agree -- Lisa, we can't know whether your diagram was incorrect unless you post it! :P
I host free online workshop/Q&A sessions called Zen and the Art of LSAT. You can find upcoming dates here: http://www.manhattanlsat.com/zen-and-the-art.cfm
User avatar
 
chewbacca
Thanks Received: 0
Vinny Gambini
Vinny Gambini
 
Posts: 2
Joined: August 05th, 2011
 
 
 

Re: Q3

by chewbacca Thu Mar 01, 2012 6:33 pm

Does anyone know a quick method to solve this problem? I used the attached diagram and then determined the following possible positions:

1st: R or Z
2nd/3rd: Z, T, or W
4th: V
5th/6th: W, S, or Y
7th: Z
8th: Q

Finally, I eliminated answer choices.

The diagram took less than a minute, but the rest of the steps took about four minutes to complete. Is there a more efficient method?
Attachments
PT38S2G1.pdf
(9.09 KiB) Downloaded 253 times
 
timmydoeslsat
Thanks Received: 887
Atticus Finch
Atticus Finch
 
Posts: 1136
Joined: June 20th, 2011
 
This post thanked 1 time.
 
trophy
Most Thanked
trophy
First Responder
 

Re: Q3

by timmydoeslsat Thu Mar 01, 2012 7:50 pm

Your global diagram is excellent.

In question 3 we are given a local rule of Z being placed 7th.

On our initial global diagram, we could have taken note that only 2 possible variables could be placed first, either R or Z. The last variable could be any one of W, Y, S, or Q. Sometimes it can be beneficial to think about those things before heading into the questions.

After placing Z 7th, I know that Q must be 8th. I also reconstructed my global diagram into a more local one, it took about 5 seconds.

I came up with this:

Image

I now also realize that of course R must be first. I noticed that in your post above you had Z going 7th but also as a possibility for going 1st, which of course is impossible as the variables are used exactly once.

This is how many diagram looked before I headed to the answer choices:

Image

Answer choices for this could be true question:

A) No, R is 1st.
B) Not enough room for T to be 4th
C) Of course this could be true. W is a floater and could go anywhere in spots 2-6.
D) Not enough room for V to be 6th.
E) No, Q is 8th.
User avatar
 
chewbacca
Thanks Received: 0
Vinny Gambini
Vinny Gambini
 
Posts: 2
Joined: August 05th, 2011
 
 
 

Re: Q3

by chewbacca Thu Mar 01, 2012 8:58 pm

Thank you for the awesome explanation, Timmy. I worked on this problem multiple times but didn't even realize that I duplicated Z in the ordering, so that was a good catch.

For answer choices B and D, would these be reasonable justifications for eliminating them?

B) No, the latest T can be is third.
D) No, the latest V can be is fourth.
 
timmydoeslsat
Thanks Received: 887
Atticus Finch
Atticus Finch
 
Posts: 1136
Joined: June 20th, 2011
 
This post thanked 1 time.
 
trophy
Most Thanked
trophy
First Responder
 

Re: Q3

by timmydoeslsat Thu Mar 01, 2012 9:22 pm

Not only are those reasonable but those are the exact reasons why those are incorrect! Good job.